Gå til innhold

Matte i media og forskning.


rlz

Anbefalte innlegg

  • 2 uker senere...
Videoannonse
Annonse

Nå har jeg tenkt og forsøkt på disse oppgavene en god stund, men jeg klarer ikke å finne noen løsning på dem. Kanskje noen her kan komme med noen kvalifiserte "gjettninger"?

 

3-6-11-18-29-42-?

Her så var det første som slo meg dette:

+3, +5, +7, +(9), +11, +13

Så jeg lurer på om det kanskje er det at differansen øker med 2 for hver, men at den for hver 4. øker med 4. Det er altså her jeg er nå, og finner ingen annen løsning.

 

 

3-5-8-13-22-39-?

 

+2, +3, +5, +9, +17

 

Her tror jeg kanskje at vi sitter på en n*2 -1, men føler meg fortsatt usikker.

Lenke til kommentar
3-6-11-18-29-42-?

Her så var det første som slo meg dette:

+3, +5, +7, +(9), +11, +13

Så jeg lurer på om det kanskje er det at differansen øker med 2 for hver, men at den for hver 4. øker med 4. Det er altså her jeg er nå, og finner ingen annen løsning.

Det virker som den ikke øker med alternerende +2 og +4, men med neste primtall.

+3, +5, +7, +11, +13, +17, +19, +23, +29, osv.

 

3-5-8-13-22-39-?

 

+2, +3, +5, +9, +17

 

Her tror jeg kanskje at vi sitter på en n*2 -1, men føler meg fortsatt usikker.

9342923[/snapback]

Det virker som *2-1, *2-2, *2-3, *2-4, *2-5 osv.

3

2*3-1 = 5

2*5-2 = 8

2*8-3 = 13

2*13-4 = 22

2*22-5 = 39

2*39-6 = 72

osv

Lenke til kommentar

Et lite matteproblem her :)

 

Kan dere løse denne her for meg?

 

en stafettløper tilbakelegger strekningen s(t ) = (5,0 m/s) x t + (0,10 m/s i andre) x t i andre på tiden t etter at han får stafettpinnen

 

Hvor langt kommer han på 10 sec?

 

 

Jeg vet ikke helt hva jeg gjør feil, men når jeg prøver, så får jeg feil. Hadde satt pris om dere kunne tatt med utregningen og.

Lenke til kommentar
hvis den oppgaven virkelig er så lett som den ser ut til skal det da bare være å sette inn t = 10 i funksjonen, altså finne s(10).

9370794[/snapback]

 

Det jeg gjorde, jeg trodde den var lett, men tydeligvis fikk jeg feil..

 

EDIT: Nå fikk jeg 50.1 meter. Er det riktig? Det fikk jeg før og, men en i klassen stod på sitt og sa det var feil :p

Endret av _Zeke
Lenke til kommentar
s(t ) = (5,0 m/s) x t + (0,10 m/s i andre) x t i andre

9370766[/snapback]

s(10) = (5 m/s)*10s + 1/2*(0,10 m/s^2)*(10 s)^2

9370877[/snapback]

Hvis det stemmer så er oppgaven feilsitert. Det står ikke noe om 1/2* i ligninga til _Zeke selv om det logisk stemmer med den klassiske mekanikken fra 2FY.

 

Akselerasjonsleddet er uansett veldig lite så det er derfor det blir i nærheten av riktig likevel. (50,5 i stedet for 51,0)

Lenke til kommentar
s(t ) = (5,0 m/s) x t + (0,10 m/s i andre) x t i andre

9370766[/snapback]

s(10) = (5 m/s)*10s + 1/2*(0,10 m/s^2)*(10 s)^2

9370877[/snapback]

Hvis det stemmer så er oppgaven feilsitert. Det står ikke noe om 1/2* i ligninga til _Zeke selv om det logisk stemmer med den klassiske mekanikken fra 2FY.

 

Akselerasjonsleddet er uansett veldig lite så det er derfor det blir i nærheten av riktig likevel. (50,5 i stedet for 51,0)

9372006[/snapback]

 

Da tror jeg oppgaven er feilsitert. Under har jeg skrevet akkurat det som står i fysikkboka (som er ny i år, men jeg tror det er 1. utkast, så litt feil er det nok)

 

1.23: En stafettløper tilbakelegger strekningen s(t) = (5.0 m/s) * t + (0.10 m/s^2) * t^2 på tiden t etter han får stafettpinnen.

 

a) Hvor langt kommer løperen på 10 sekunder? Hva er gjennomsnittsfarten i denne tiden?

 

Dere trenger ikke å løse den. Det har jeg gjort selv ;)

Lenke til kommentar

Det er et "magisk triks" som omformer en ligning til noe helt annet. Hensikten er at den nye formen skal være lettere å løse. Etter at det er løst i omformet tilstand må det omformes tilbake. På denne måten kan man løse oppgaver på en lettere måte. Det er i hvert fall svaret på hvorfor vi gjør det.

 

Forklaringen på hvordan Laplace-transformasjonen ble oppfunnet og hva det egentlig er har jeg ikke skjønt fullt ut og kommer vel neppe til å gjøre det noen gang heller. Det eneste jeg kan si der er at det fungerer men jeg vet ikke hvorfor.

 

Redigert: http://en.wikipedia.org/wiki/Laplace_transform

 

In actual physical systems the Laplace transform is often interpreted as a transformation from the time-domain point of view, in which inputs and outputs are understood as functions of time, to the frequency-domain point of view, where the same inputs and outputs are seen as functions of complex angular frequency, or radians per unit time. This transformation not only provides a fundamentally different way to understand the behavior of the system, but it also drastically reduces the complexity of the mathematical calculations required to analyze the system.
Lenke til kommentar

Vil vise:

lim_{x->sqrt(3)} 1/x^2 = 1/3.

 

Vi har |x-sqrt(3)| < d, og skal se om |1/x^2 - 1/3| < e.

|1/x^2 - 1/3| = |(3-x^2)/(3x^2)| = 1/3 |(3-x^2)/x^2|

= 1/3 |3-x^2|/x^2 = 1/3 |x^2-3|/x^2 <= 1/9 |x^2 - 3|.

 

Siden |x-sqrt(3)| < 1 < d, må også

|x-sqrt(3)|^2 = |x^2 - 2sqrt(3)x + 3| <= |x^2 - 3| < d.

Hvis vi så setter d = 9e, får vi:

1/9 |x^2 - 3| < 1/9 d = 1/9 * 9e = e, og vi er ferdige.

 

 

Dette ble veldig grovt, og det er noen små ting som burde argumenteres bedre, men det funker som en liten skisse.

Lenke til kommentar

Opprett en konto eller logg inn for å kommentere

Du må være et medlem for å kunne skrive en kommentar

Opprett konto

Det er enkelt å melde seg inn for å starte en ny konto!

Start en konto

Logg inn

Har du allerede en konto? Logg inn her.

Logg inn nå
  • Hvem er aktive   0 medlemmer

    • Ingen innloggede medlemmer aktive
×
×
  • Opprett ny...